K
Khách

Hãy nhập câu hỏi của bạn vào đây, nếu là tài khoản VIP, bạn sẽ được ưu tiên trả lời.

6 tháng 3 2016

tớ ko bít. Giúp với. Nhé

6 tháng 3 2016

Áp dụng bất đẳng thức:\(\frac{1}{x}+\frac{1}{y}+\frac{1}{z}\ge\frac{9}{x+y+z}\)

ta có:

\(A=\frac{1}{\left(a-b\right)^2}+\frac{1}{\left(b-c\right)^2}+\frac{1}{\left(c-a\right)^2}\ge\frac{9}{\left(a-b\right)^2+\left(b-c\right)^2+\left(c-a\right)^2}\)

Đến đâu Cm dưới mẫu <4 nữa là đc

Tích nha

7 tháng 8 2016

Vì vai trò bình đẳng của các ẩn  \(a,b,c\)  là như nhau nên không mất tính tổng quát, ta có thể giả sử:

\(2\ge c>b>a\ge0\) \(\left(\alpha\right)\) (do  \(a,b,c\)  đôi một khác nhau nên cũng không đồng thời bằng nhau)

Áp dụng bđt  \(AM-GM\)  cho từng bộ số gồm có các số không âm, ta có:

\(\left(i\right)\)  Với  \(\frac{1}{\left(a-b\right)^2}>0;\)  \(\left[-\left(a-b\right)\right]>0\)\(\frac{1}{\left(a-b\right)^2}+\left[-\left(a-b\right)\right]+\left[-\left(a-b\right)\right]\ge3\sqrt[3]{\frac{1}{\left(a-b\right)^2}.\left[-\left(a-b\right)\right]\left[-\left(a-b\right)\right]}=3\)

\(\Rightarrow\)  \(\frac{1}{\left(a-b\right)^2}\ge3-2\left(b-a\right)\)  \(\left(1\right)\)

\(\left(ii\right)\) Với  \(\frac{1}{\left(b-c\right)^2}>0;\) \(\left[-\left(b-c\right)\right]>0\)

 \(\frac{1}{\left(b-c\right)^2}+\left[-\left(b-c\right)\right]+\left[-\left(b-c\right)\right]\ge3\sqrt[3]{\frac{1}{\left(b-c\right)^2}.\left[-\left(b-c\right)\right]\left[-\left(b-c\right)\right]}=3\)

\(\Rightarrow\)  \(\frac{1}{\left(b-c\right)^2}\ge3-2\left(c-b\right)\)  \(\left(2\right)\)

\(\left(iii\right)\)  Với  \(\frac{1}{\left(c-a\right)^2}>0;\)  \(\frac{c-a}{16}>0\)

\(\frac{1}{\left(c-a\right)^2}+\frac{c-a}{8}+\frac{c-a}{8}\ge3\sqrt[3]{\frac{1}{\left(c-a\right)^2}.\frac{\left(c-a\right)}{8}.\frac{\left(c-a\right)}{8}}=\frac{3}{4}\)

\(\Rightarrow\)  \(\frac{1}{\left(c-a\right)^2}\ge\frac{3}{4}-\frac{c-a}{4}\)  \(\left(3\right)\)

Cộng từng vế ba bất đẳng thức  \(\left(1\right);\)  \(\left(2\right)\)  và   \(\left(3\right)\)  , ta được:

\(\frac{1}{\left(a-b\right)^2}+\frac{1}{\left(b-c\right)^2}+\frac{1}{\left(c-a\right)^2}\ge3-2\left(b-a\right)+3-2\left(c-b\right)+\frac{3}{4}-\frac{c-a}{4}\)

nên   \(\frac{1}{\left(a-b\right)^2}+\frac{1}{\left(b-c\right)^2}+\frac{1}{\left(c-a\right)^2}\ge\frac{27}{4}-\frac{9\left(c-a\right)}{4}=\frac{27}{4}+\frac{9\left(a-c\right)}{4}\)

Mặt khác, từ  \(\left(\alpha\right)\)  ta suy ra được:  \(\hept{\begin{cases}a\ge0\\2\ge c\end{cases}}\)

nên   \(a+2\ge c\) hay nói cách khác  \(a-c\ge-2\)

Do đó,  \(\frac{1}{\left(a-b\right)^2}+\frac{1}{\left(b-c\right)^2}+\frac{1}{\left(c-a\right)^2}\ge\frac{27}{4}+\frac{9.\left(-2\right)}{4}=\frac{9}{4}\)

Dấu  \("="\)  xảy ra khi và chỉ khi  \(\hept{\begin{cases}a=0\\b=1\\c=2\end{cases}}\)  (thỏa mãn  \(\left(\alpha\right)\)  )

7 tháng 8 2016

Vì vai trò bình đẳng của các ẩn  \(a,b,c\)  là như nhau nên không mất tính tổng quát, ta có thể giả sử:

\(2\ge c>b>a\ge0\) \(\left(\alpha\right)\) (do  \(a,b,c\)  đôi một khác nhau nên cũng không đồng thời bằng nhau)

Áp dụng bđt  \(AM-GM\)  cho từng bộ số gồm có các số không âm, ta có:

\(\left(i\right)\)  Với  \(\frac{1}{\left(a-b\right)^2}>0;\)  \(\left[-\left(a-b\right)\right]>0\)\(\frac{1}{\left(a-b\right)^2}+\left[-\left(a-b\right)\right]+\left[-\left(a-b\right)\right]\ge3\sqrt[3]{\frac{1}{\left(a-b\right)^2}.\left[-\left(a-b\right)\right]\left[-\left(a-b\right)\right]}=3\)

\(\Rightarrow\)  \(\frac{1}{\left(a-b\right)^2}\ge3-2\left(b-a\right)\)  \(\left(1\right)\)

\(\left(ii\right)\) Với  \(\frac{1}{\left(b-c\right)^2}>0;\) \(\left[-\left(b-c\right)\right]>0\)

 \(\frac{1}{\left(b-c\right)^2}+\left[-\left(b-c\right)\right]+\left[-\left(b-c\right)\right]\ge3\sqrt[3]{\frac{1}{\left(b-c\right)^2}.\left[-\left(b-c\right)\right]\left[-\left(b-c\right)\right]}=3\)

\(\Rightarrow\)  \(\frac{1}{\left(b-c\right)^2}\ge3-2\left(c-b\right)\)  \(\left(2\right)\)

\(\left(iii\right)\)  Với  \(\frac{1}{\left(c-a\right)^2}>0;\)  \(\frac{c-a}{16}>0\)

\(\frac{1}{\left(c-a\right)^2}+\frac{c-a}{8}+\frac{c-a}{8}\ge3\sqrt[3]{\frac{1}{\left(c-a\right)^2}.\frac{\left(c-a\right)}{8}.\frac{\left(c-a\right)}{8}}=\frac{3}{4}\)

\(\Rightarrow\)  \(\frac{1}{\left(c-a\right)^2}\ge\frac{3}{4}-\frac{c-a}{4}\)  \(\left(3\right)\)

Cộng từng vế ba bất đẳng thức  \(\left(1\right);\)  \(\left(2\right)\)  và   \(\left(3\right)\)  , ta được:

\(\frac{1}{\left(a-b\right)^2}+\frac{1}{\left(b-c\right)^2}+\frac{1}{\left(c-a\right)^2}\ge3-2\left(b-a\right)+3-2\left(c-b\right)+\frac{3}{4}-\frac{c-a}{4}\)

nên   \(\frac{1}{\left(a-b\right)^2}+\frac{1}{\left(b-c\right)^2}+\frac{1}{\left(c-a\right)^2}\ge\frac{27}{4}-\frac{9\left(c-a\right)}{4}=\frac{27}{4}+\frac{9\left(a-c\right)}{4}\)

Mặt khác, từ  \(\left(\alpha\right)\)  ta suy ra được:  \(\hept{\begin{cases}a\ge0\\2\ge c\end{cases}}\)

nên   \(a+2\ge c\) hay nói cách khác  \(a-c\ge-2\)

Do đó,  \(\frac{1}{\left(a-b\right)^2}+\frac{1}{\left(b-c\right)^2}+\frac{1}{\left(c-a\right)^2}\ge\frac{27}{4}+\frac{9.\left(-2\right)}{4}=\frac{9}{4}\)

Dấu  \("="\)  xảy ra khi và chỉ khi  \(a=0;b=1;c=2\)  (thỏa mãn  \(\left(\alpha\right)\)  )

28 tháng 8 2020

Áp dụng giả thiết và một đánh giá quen thuộc, ta được: \(16\left(a+b+c\right)\ge\frac{1}{a}+\frac{1}{b}+\frac{1}{c}=\frac{ab+bc+ca}{abc}=\frac{\left(ab+bc+ca\right)^2}{abc\left(ab+bc+ca\right)}\ge\frac{3\left(a+b+c\right)}{ab+bc+ca}\)hay \(\frac{1}{6\left(ab+bc+ca\right)}\le\frac{8}{9}\)

Đến đây, ta cần chứng minh \(\frac{1}{\left(a+b+\sqrt{2\left(a+c\right)}\right)^3}+\frac{1}{\left(b+c+\sqrt{2\left(b+a\right)}\right)^3}+\frac{1}{\left(c+a+\sqrt{2\left(c+b\right)}\right)^3}\le\frac{1}{6\left(ab+bc+ca\right)}\)

 Áp dụng bất đẳng thức Cauchy cho ba số dương ta có \(a+b+\sqrt{2\left(a+c\right)}=a+b+\sqrt{\frac{a+c}{2}}+\sqrt{\frac{a+c}{2}}\ge3\sqrt[3]{\frac{\left(a+b\right)\left(a+c\right)}{2}}\)hay \(\left(a+b+\sqrt{2\left(a+c\right)}\right)^3\ge\frac{27\left(a+b\right)\left(a+c\right)}{2}\Leftrightarrow\frac{1}{\left(a+b+2\sqrt{a+c}\right)^3}\le\frac{2}{27\left(a+b\right)\left(a+c\right)}\)

Hoàn toàn tương tự ta có \(\frac{1}{\left(b+c+2\sqrt{b+a}\right)^3}\le\frac{2}{27\left(b+c\right)\left(b+a\right)}\)\(\frac{1}{\left(c+a+2\sqrt{c+b}\right)^3}\le\frac{2}{27\left(c+a\right)\left(c+b\right)}\)

Cộng theo vế các bất đẳng thức trên ta được \(\frac{1}{\left(a+b+\sqrt{2\left(a+c\right)}\right)^3}+\frac{1}{\left(b+c+\sqrt{2\left(b+a\right)}\right)^3}+\frac{1}{\left(c+a+\sqrt{2\left(c+b\right)}\right)^3}\le\frac{4\left(a+b+c\right)}{27\left(a+b\right)\left(b+c\right)\left(c+a\right)}\)Phép chứng minh sẽ hoàn tất nếu ta chỉ ra được \(\frac{4\left(a+b+c\right)}{27\left(a+b\right)\left(b+c\right)\left(c+a\right)}\le\frac{1}{6\left(ab+bc+ca\right)}\)\(\Leftrightarrow\left(a+b\right)\left(b+c\right)\left(c+a\right)\ge\frac{8}{9}\left(ab+bc+ca\right)\left(a+b+c\right)\)

Đây là một đánh giá đúng, thật vậy: đặt a + b + c = p; ab + bc + ca = q; abc = r thì bất đẳng thức trên trở thành \(pq-r\ge\frac{8}{9}pq\Leftrightarrow\frac{1}{9}pq\ge r\)*đúng vì \(a+b+c\ge3\sqrt[3]{abc}\)\(ab+bc+ca\ge3\sqrt[3]{\left(abc\right)^2}\))

Vậy bất đẳng thức được chứng minh

Đẳng thức xảy ra khi \(a=b=c=\frac{1}{4}\)

2 tháng 12 2020

Đặt \(\left(\frac{1}{a},\frac{1}{b},\frac{1}{c}\right)=\left(x,y,z\right)\)

\(x+y+z\ge\frac{x^2+2xy}{2x+y}+\frac{y^2+2yz}{2y+z}+\frac{z^2+2zx}{2z+x}\)

\(\Leftrightarrow x+y+z\ge\frac{3xy}{2x+y}+\frac{3yz}{2y+z}+\frac{3zx}{2z+x}\)

\(\frac{3xy}{2x+y}\le\frac{3}{9}xy\left(\frac{1}{x}+\frac{1}{x}+\frac{1}{y}\right)=\frac{1}{3}\left(x+2y\right)\)

\(\Rightarrow\Sigma_{cyc}\frac{3xy}{2x+y}\le\frac{1}{3}\left[\left(x+2y\right)+\left(y+2z\right)+\left(z+2x\right)\right]=x+y+z\)

Dấu "=" xảy ra khi x=y=z

8 tháng 8 2017

bài này mà giải theo SOS là hơi bị tuyệt vời nhé =)))

8 tháng 8 2017

em moi co lop 7

11 tháng 4 2017

\(\frac{a^3}{\left(1+b\right)\left(1+c\right)}+\frac{b^3}{\left(1+c\right)\left(1+a\right)}+\frac{c^3}{\left(1+a\right)\left(1+b\right)}\)

Ta có:

\(\frac{a^3}{\left(1+b\right)\left(1+c\right)}+\frac{1+b}{8}+\frac{1+c}{8}\ge\frac{3a}{4}\)

\(\Leftrightarrow\frac{a^3}{\left(1+b\right)\left(1+c\right)}\ge\frac{6a-b-c-2}{8}\)

Tương tự ta có: \(\hept{\begin{cases}\frac{b^3}{\left(1+c\right)\left(1+a\right)}\ge\frac{6b-c-a-2}{8}\\\frac{c^3}{\left(1+a\right)\left(1+b\right)}\ge\frac{6c-a-b-2}{8}\end{cases}}\)

Cộng vế theo vế ta được

\(\frac{a^3}{\left(1+b\right)\left(1+c\right)}+\frac{b^3}{\left(1+c\right)\left(1+a\right)}+\frac{c^3}{\left(1+a\right)\left(1+b\right)}\ge\frac{6a-b-c-2}{8}+\frac{6b-c-a-2}{8}+\frac{6c-a-b-2}{8}\)

\(=\frac{a+b+c}{2}-\frac{3}{4}\ge\frac{3}{2}.\sqrt[3]{abc}-\frac{3}{4}=\frac{3}{2}-\frac{3}{4}=\frac{3}{4}\)

10 tháng 4 2017

Mai mình làm cho

1 tháng 1 2021

giả sử \(a\ge b\ge c\ge0\)

Ta có: \(a+\frac{b}{2}-\frac{a^2+ab+b^2}{a+b}=\frac{1}{2}\left(ab-b^2\right)\ge0\Rightarrow a+\frac{b}{2}\ge\frac{a^2+ab+b^2}{a+b}\)

\(b+\frac{a}{2}-\frac{a^2+ab+b^2}{a+b}=\frac{1}{2}\left(ab-a^2\right)\le0\Rightarrow b+\frac{a}{2}\le\frac{a^2+ab+b^2}{a+b}\)

Tương tự: \(b+\frac{c}{2}\ge\frac{b^2+bc+c^2}{b+c}\ge c+\frac{b}{2};a+\frac{c}{2}\ge\frac{a^2+ac+c^2}{a+c}\ge c+\frac{a}{2}\)

Lại có:+) \(\frac{a^3-b^3}{a+b}+\frac{b^3-c^3}{b+c}+\frac{c^3-a^3}{c+a}\)

\(=\left(a-b\right)\frac{a^2+ab+b^2}{a+b}+\left(b-c\right)\frac{b^2+bc+c^2}{b+c}-\left(a-c\right)\frac{a^2+ac+c^2}{a+c}\)

\(\ge\left(a-b\right)\left(b+\frac{a}{2}\right)+\left(b-c\right)\left(c+\frac{a}{2}\right)-\left(a-c\right)\left(a+\frac{c}{2}\right)\)

\(\ge\frac{-1}{4}\left[\left(a-b\right)^2+\left(b-c\right)^2+\left(c-a\right)^2\right]\left(1\right)\)

+) \(\frac{a^3-b^3}{a+b}+\frac{b^3-c^3}{b+c}+\frac{c^3-a^3}{c+a}\)

\(=\left(a-b\right)\frac{a^2+ab+b^2}{a+b}+\left(b-c\right)\frac{b^2+bc+c^2}{b+c}-\left(a-c\right)\frac{a^2+ac+c^2}{a+c}\)

\(\le\left(a-b\right)\left(a+\frac{b}{2}\right)+\left(b-c\right)\left(b+\frac{c}{2}\right)-\left(a-c\right)\left(c+\frac{a}{2}\right)\)

\(\le\frac{1}{4}\left[\left(a-b\right)^2+\left(b-c\right)^2+\left(c-a\right)^2\right]\left(2\right)\)

Từ 1,2 => đpcm

2 tháng 1 2021

BĐT đã cho tuong duong voi:

\(\left|\frac{\left(a-b\right)\left(b-c\right)\left(c-a\right)\left(ab+bc+ca\right)}{\left(a+b\right)\left(b+c\right)\left(c+a\right)}\right|\le\frac{1}{4}\left[\Sigma\left(a-b\right)^2\right]\)

Theo AM-GM ta có: \(\left(ab+bc+ca\right)\le\frac{9}{8}\cdot\frac{\left(a+b\right)\left(b+c\right)\left(c+a\right)}{a+b+c}\)

Có: \(VT\le\frac{9}{8}\left|\frac{\sqrt{\left(a-b\right)^2\left(b-c\right)^2\left(c-a\right)^2}}{\left(a+b+c\right)}\right|=\frac{9\sqrt{\left(a-b\right)^2\left(b-c\right)^2\left(c-a\right)^2}}{8\left(a+b+c\right)}\)

Cần chứng minh: \(4\left(a+b+c\right)^2\left[\Sigma\left(a-b\right)^2\right]^2\ge9\left(a-b\right)^2\left(b-c\right)^2\left(c-a\right)^2\)

Rõ ràng \(\Sigma\left(a-b\right)^2\ge3\sqrt[3]{\left(a-b\right)^2\left(b-c\right)^2\left(c-a\right)^2}\)

Cần cm: \(36\left(a+b+c\right)^2\sqrt[3]{\left(a-b\right)^4\left(b-c\right)^4\left(c-a\right)^4}\ge9\sqrt[3]{\left(a-b\right)^6\left(b-c\right)^6\left(c-a\right)^6}\)

Hay \(4\left(a+b+c\right)^2\ge\sqrt[3]{\left(a-b\right)^2\left(b-c\right)^2\left(c-a\right)^2}\)

Tiếp tục là điều hiển nhiên do \(VT\ge4\left[\left(a+b+c\right)^2-3\left(ab+bc+ca\right)\right]\)

\(=2\left[\left(a-b\right)^2+\left(b-c\right)^2+\left(c-a\right)^2\right]\)

\(\ge6\sqrt[3]{\left(a-b\right)^2\left(b-c\right)^2\left(c-a\right)^2}\ge VP\)

Đẳng thức xảy ra khi \(\hept{\begin{cases}\left(a-b\right)\left(b-c\right)\left(c-a\right)=0\\a-b=b-c=c-a\\a=b=c\end{cases}}\Leftrightarrow a=b=c.\)